Home / Logical Reasoning / Statement and Argument :: Discussion

Discussion :: Statement and Argument

  1. Directions : Each of the following question consists of a statement followed by two arguments I and II. You have to decide which of the arguments is a STRONG arguments and which is a WEAK Argument.

    Statement : Should the railways immediately stop issuing free passes to all its employees?

    Arguments :
    I. No. The employees have the right to travel free.
    II. Yes. This will help railways to provide better facility.

  2. A.

     Only argument I is strong

    B.

     Only argument II is strong

    C.

     Either I or II is strong

    D.

     Neither I nor II is strong

    E.

     Both I and II are strong

    View Answer

    Workspace

    Answer : Option D

    Explanation :

    The free passes given to railway employees is a privilege for them, not their right. So, argument I does not hold. Argument II seems to be vague.


Be The First To Comment